GMAT Prep - Scanners

This topic has expert replies
User avatar
Master | Next Rank: 500 Posts
Posts: 355
Joined: Thu Feb 19, 2009 12:42 pm
Thanked: 2 times
Followed by:1 members

GMAT Prep - Scanners

by vineetbatra » Wed Oct 21, 2009 6:41 am
I will post the OA later, but can someone explain what is the conclusion in this argument also, what does "A questionable assumption" type of question really look for:

The general availability of high-quality electronic scanners and color printers for computers has made the counterfeiting of checks much easier. In order to deter such counterfeiting, several banks plan to issue to their corporate customers checks that contain dots too small to be accurately duplicated by any electronic scanner currently available; when such checks are scanned and printed, the dots seem to blend together in such a way that the word "VOID" appears on the check

A questionable assumption of the plan is that

A. in the territory served by the banks the proportion of counterfeit checks that are made using electronic scanners has remained approximately constant over the past few years
B. most counterfeiters who use electronic scanners counterfeit checks only for relatively large amounts of money
C. the smallest dots on the proposed checks cannot be distinguished visually except under strong magnification
D. most corporations served by these banks will not have to pay more for the new checks than for traditional checks
E. the size of the smallest dots that generally available electronic scanners are able to reproduce accurately will not decrease significantly in the near future.

Senior | Next Rank: 100 Posts
Posts: 80
Joined: Wed Jun 04, 2008 1:16 pm
Thanked: 3 times

Re: GMAT Prep - Scanners

by mpaudena » Wed Oct 21, 2009 10:58 am
IMO the answer is E.

E says that the argument relies on the assumption that the printers will not be able to print smaller dots accurately in the near future.

vineetbatra wrote: E. the size of the smallest dots that ... scanners are able to reproduce accurately will not decrease significantly in the near future.
if the size that the scanners are accurately able to produce decreases then they are able to reproduce smaller sized dots thereby circumventing the efforts of the banks.

I think "questionable assumption" questions should be treated just like assumption questions. It's questionability is irrelevant so long as it is the assumption on which the argument relies on. I suppose this assumption is questionable because it is not likely that the generally available printers will suddenly improve. It is more likely that their will be improved scanners that are brought into the market.

Senior | Next Rank: 100 Posts
Posts: 35
Joined: Tue Sep 01, 2009 4:18 pm

by [email protected] » Thu Oct 22, 2009 3:59 am
Should be the following

D. most corporations served by these banks will not have to pay more for the new checks than for traditional checks

It is assumed that the cost of the transfer to the new system is not going to have additional costs...This is a questionable assumption...

And E seems to more implied from the last premise....

can you post the OA pl....

Senior | Next Rank: 100 Posts
Posts: 73
Joined: Sat Oct 03, 2009 10:37 pm

by kiennguyen » Thu Oct 22, 2009 5:44 am
OA please!

Master | Next Rank: 500 Posts
Posts: 177
Joined: Thu Aug 14, 2008 11:59 am
Thanked: 25 times

by mp2437 » Thu Oct 22, 2009 6:41 am
OA: E

User avatar
Master | Next Rank: 500 Posts
Posts: 113
Joined: Sat Oct 17, 2009 10:51 am
Thanked: 10 times
GMAT Score:690

User avatar
Master | Next Rank: 500 Posts
Posts: 355
Joined: Thu Feb 19, 2009 12:42 pm
Thanked: 2 times
Followed by:1 members

Re: GMAT Prep - Scanners

by vineetbatra » Thu Oct 22, 2009 8:24 am
mpaudena wrote:IMO the answer is E.

if the size that the scanners are accurately able to produce decreases then they are able to reproduce smaller sized dots thereby circumventing the efforts of the banks.
Thanks for the response the OA is E, and I selected E because no other answer was good enough; however I still have a doubt.

E says that the Size of the smallest dots generally available electronic scanners are able to reproduce accurately will not decrease

If the size itself will not decrease then it kindda contradicts the stimulus in which it says that the size of the dots will decrease. This is the part which I am not able to comprehend.

Senior | Next Rank: 100 Posts
Posts: 80
Joined: Wed Jun 04, 2008 1:16 pm
Thanked: 3 times

Re: GMAT Prep - Scanners

by mpaudena » Thu Oct 22, 2009 10:00 am
It does sort of contradict and that is the reason it is a questionable assumption. Here is what the stimulus says:
vineetbatra wrote: .... several banks plan to issue to their corporate customers checks that contain dots too small to be accurately duplicated by any electronic scanner currently available....
The scanners currently available cannot accurately duplicate the dots on the new checks...

E says that the argument assumes that the ability of the scanners to duplicate dots will not improve. If the size of the dots the currently available scanners can duplicate is reduced then the banks efforts are thwarted. So the assumption is that the size will not get smaller.

Legendary Member
Posts: 2326
Joined: Mon Jul 28, 2008 3:54 am
Thanked: 173 times
Followed by:2 members
GMAT Score:710

Re: GMAT Prep - Scanners

by gmatmachoman » Fri Oct 23, 2009 1:15 pm
mpaudena wrote:It does sort of contradict and that is the reason it is a questionable assumption. Here is what the stimulus says:
vineetbatra wrote: .... several banks plan to issue to their corporate customers checks that contain dots too small to be accurately duplicated by any electronic scanner currently available....
The scanners currently available cannot accurately duplicate the dots on the new checks...

E says that the argument assumes that the ability of the scanners to duplicate dots will not improve. If the size of the dots the currently available scanners can duplicate is reduced then the banks efforts are thwarted. So the assumption is that the size will not get smaller.
Nice reply!!

Does Questionable assumption is it of the format "All the below are all assumptions EXCEPT"??

Please share ur input.

Senior | Next Rank: 100 Posts
Posts: 73
Joined: Sat Oct 03, 2009 10:37 pm

good reasoning!

by kiennguyen » Fri Oct 23, 2009 7:32 pm
thanks, mpaudena!

Junior | Next Rank: 30 Posts
Posts: 21
Joined: Mon Jul 20, 2009 11:57 pm

by ReyWilli » Sat Oct 31, 2009 10:30 pm
I made the mistake of elimating E because it said "....in the future"

I thought well, the future isnt mentioned anywhere in the stimulus so its irrelevant. I think I should have looked at the overall meaning.

Thanks for the explanations
GMATJunky

GMAT Instructor
Posts: 1302
Joined: Mon Oct 19, 2009 2:13 pm
Location: Toronto
Thanked: 539 times
Followed by:164 members
GMAT Score:800

by Testluv » Sat Oct 31, 2009 10:49 pm
Does Questionable assumption is it of the format "All the below are all assumptions EXCEPT"??
Hi gmatmachoman,

Definitely not. A "questionable assumption" is the same as any plain jane necessary assumption question. For the bank's technology to work, generally available scanners can't get better at scanning at small dots. If general scanners got better at scanning small dots (that is, if the size of the dots they are able to scan decreased), then the bank's technology/plan may fail. As such E is a necessary assumption for the argument.
Kaplan Teacher in Toronto

Legendary Member
Posts: 2326
Joined: Mon Jul 28, 2008 3:54 am
Thanked: 173 times
Followed by:2 members
GMAT Score:710

by gmatmachoman » Fri Nov 06, 2009 9:23 am
Testluv wrote:
Does Questionable assumption is it of the format "All the below are all assumptions EXCEPT"??
Hi gmatmachoman,

Definitely not. A "questionable assumption" is the same as any plain jane necessary assumption question. For the bank's technology to work, generally available scanners can't get better at scanning at small dots. If general scanners got better at scanning small dots (that is, if the size of the dots they are able to scan decreased), then the bank's technology/plan may fail. As such E is a necessary assumption for the argument.

Many Thx @Testluv,

I understand that one now...

Senior | Next Rank: 100 Posts
Posts: 88
Joined: Mon Oct 11, 2010 1:20 am
Thanked: 1 times

by Onell » Fri May 06, 2011 12:33 am
Testluv wrote:
Does Questionable assumption is it of the format "All the below are all assumptions EXCEPT"??
Hi gmatmachoman,

Definitely not. A "questionable assumption" is the same as any plain jane necessary assumption question. For the bank's technology to work, generally available scanners can't get better at scanning at small dots. If general scanners got better at scanning small dots (that is, if the size of the dots they are able to scan decreased), then the bank's technology/plan may fail. As such E is a necessary assumption for the argument.
The argument says contain dots too small to be accurately duplicated by any electronic scanner currently available; .... It talks only about the currently available scanner..furthermore the argument is open to the possibility that the scanner can get better in future and it doesn't say the problem will be gone forever.... so shouldn't option E be out of scope?

Am I missing sth?

User avatar
Senior | Next Rank: 100 Posts
Posts: 42
Joined: Mon Oct 18, 2010 11:39 pm

by novel » Sat May 07, 2011 1:01 am
Onell wrote:
Testluv wrote:
Does Questionable assumption is it of the format "All the below are all assumptions EXCEPT"??
Hi gmatmachoman,

Definitely not. A "questionable assumption" is the same as any plain jane necessary assumption question. For the bank's technology to work, generally available scanners can't get better at scanning at small dots. If general scanners got better at scanning small dots (that is, if the size of the dots they are able to scan decreased), then the bank's technology/plan may fail. As such E is a necessary assumption for the argument.
The argument says contain dots too small to be accurately duplicated by any electronic scanner currently available; .... It talks only about the currently available scanner..furthermore the argument is open to the possibility that the scanner can get better in future and it doesn't say the problem will be gone forever.... so shouldn't option E be out of scope?

Am I missing sth?


According to argument it possible in future.However it has to be assumed that high quality scanner will not be available in future to conclude that counterfeiting can be avoided.